Вы находитесь на странице: 1из 39

EE 369

POWER SYSTEM ANALYSIS

Lecture 16
Economic Dispatch
Tom Overbye and Ross Baldick

1
Announcements
Read Chapters 6 (section 6.12) and 7
(sections 7.1 to 7.3).
Homework 12 is 6.62, 6.63, 6.67
(calculate economic dispatch for values
of load from 55 MW to 350 MW); due
Tuesday, 11/29.
Class review and course evaluation on
Tuesday, 11/29.
Midterm III on Thursday, 12/1, including
material through Homework 12.
2
Economic Dispatch:
Formulation
The goal of economic dispatch is to determine
the generation dispatch that minimizes the
instantaneous operating cost, subject to the
constraint that total generation = total load +
losses

m
Minimize CT @ Ci ( PGi )
i 1
Initially we'll
Such that ignore generato
m limits and the
PGi PD PLosses losses
i 1 3
Unconstrained Minimization
This is a minimization problem with a single
equality constraint
For an unconstrained minimization a
necessary (but not sufficient) condition for a
minimum is the gradient of the function must
be zero,
The gradient generalizes
f (x) 0 the first derivative
for multi-variable problems:

f (x) f (x) f (x)


f (x) @ , ,K ,
x1 x2 x n
4
Minimization with Equality
Constraint
When the minimization is constrained with an
equality constraint we can solve the problem using
the method of Lagrange Multipliers
Key idea is to represent a constrained minimization
problem as an unconstrained problem.

That is, for the general problem


minimize f (x) s.t. g(x) 0
We define the Lagrangian L(x, ) f (x ) Tg (x )
Then a necessary condition for a minimum is the
L x(x, ) 0 and L (x , ) 0 5
Economic Dispatch
Lagrangian
For the economic dispatch we have a minimization
constrained with a single equality constraint
m m
L(PG , ) Ci ( PGi ) ( PD PGi ) (no losses)
i 1 i 1
The necessary conditions for a minimum are
L(PG , ) dCi ( PGi )
0 (for i 1 to m)
PGi dPGi
m
PD PGi 0
i 1

6
Economic Dispatch Example
What is economic dispatch for a two generator
system PD PG1 PG 2 500 MW and
C1 ( PG1 ) 1000 20 PG1 0.01PG21 $/h
C2 ( PG 2 ) 400 15PG 2 0.03PG22 $/h
Using the Lagrange multiplier method we know:
dC1 ( PG1 )
20 0.02 PG1 0
dPG1
dC2 ( PG 2 )
15 0.06 PG 2 0
dPG 2
500 PG1 PG 2 0
7
Economic Dispatch
Example, contd
We therefore need to solve three linear equations
20 0.02 PG1 0
15 0.06 PG 2 0
500 PG1 PG 2 0
0.02 0 1 P G1 20
0 0.06 1 P G 2 15

1 1 0 500
PG1 312.5 MW
P 187.5 MW
G 2
26.2 $/MWh 8
Economic dispatch example,
contd
At the solution, both generators have the same
marginal (or incremental) cost, and this
common marginal cost is equal to .
Intuition behind solution:
If marginal costs of generators were different, then
by decreasing production at higher marginal cost
generator, and increasing production at lower
marginal cost generator we could lower overall costs.
Generalizes to any number of generators.
If demand changes, then change in total costs
can be estimated from .

9
Economic dispatch example,
contd
Another way to solve the equations is to:
Rearrange the first two equations to solve for PG1
and PG2 in terms of ,
Plug into third equation and solve for ,
Use the solved value of to evaluate PG1 and PG2.
This works even when relationship between
generation levels and is more complicated:
Equations are more complicated than linear when
there are maximum and minimum generation
limits or we consider losses.

10
Lambda-Iteration Solution
Method
Discussion on previous page leads to lambda-
iteration method:
this method requires a unique mapping from a value
of lambda (marginal cost) to each generators MW
output:
PGi ( ).of lambda (common marginal cost),
for any choice
the generators collectively produce a total MW
output,
the method then starts with values of lambda below
and above the optimal value (corresponding to too
little and too much total output), and then iteratively
brackets the optimal value.

11
Lambda-Iteration Algorithm
Pick L and H such that
m m
Gi ) PD 0
P ( L
Gi ) PD 0
P ( H

i 1 i 1

While H L Do
M ( H L ) / 2
m
If Gi
P ( M
) PD 0 Then H
M

i 1

Else L M
End While 12
Lambda-Iteration: Graphical
View
In the graph shown below for each value of lam
there is a unique PGi foreachgenerator.This
relationshipisthePGi()function.

13
Lambda-Iteration Example
Consider a three generator system with
IC1 ( PG1 ) 15 0.02 PG1 $/MWh
IC2 ( PG 2 ) 20 0.01PG 2 $/MWh
IC3 ( PG 3 ) 18 0.025 PG 3 $/MWh
and with constraint PG1 PG 2 PG 3 1000MW
Rewriting generation as a function of , PGi ( ),
we have
15 20
PG1 ( ) PG2 ( )
0.02 0.01
18
PG3 ( ) 14
0.025
Lambda-Iteration Example,
m contd
L
Pick so L
PGi ( ) 1000 0 and
i=1
m
Gi ) 1000 0
P ( H

i=1
m
Try L
20 then PGi (20) 1000
i 1
15 20 18
1000 670 MW
0.02 0.01 0.025
m
Try H 30 then PGi (30) 1000 1230 MW
i 1
15
Lambda-Iteration Example,
contd
Pick convergence tolerance 0.05 $/MWh
Then iterate since H L 0.05

M ( H L ) / 2 25
m
Then since Gi
P (25) 1000 280 we set H
25
i 1
Since 25 20 0.05
M (25 20) / 2 22.5
m
Gi
P (22.5) 1000 195 we set L
22.5
i 1 16
Lambda-Iteration Example,
contd
Continue iterating until H L 0.05
*
The solution value of , , is 23.53 $/MWh
Once * is known we can calculate the PGi
23.53 15
PG1 (23.5) 426 MW
0.02
23.53 20
PG 2 (23.5) 353 MW
0.01
23.53 18
PG 3 (23.5) 221 MW
0.025
17
Thirty Bus ED Example
Case is economically dispatched (without con
the incremental impact of the system losses).

18
Generator MW Limits
Generators have limits on the minimum
and maximum amount of power they can
produce
Typically the minimum limit is not zero.
Because of varying system economics
usually many generators in a system are
operated at their maximum MW limits:
Baseload generators are at their maximum
limits except during the off-peak.

19
Lambda-Iteration with Gen
Limits

In the lambda-iteration method the limits are taken


into account when calculating PGi ( ) :
if calculated production for PGi PGi ,max
then set PGi ( ) PGi ,max
if calculated production for PGi PGi ,min
then set PGi ( ) PGi ,min

20
Lambda-Iteration Gen Limit
In the previous threeExample
generator example assume
the same cost characteristics but also with limits
0 PG1 300 MW 100 PG2 500 MW
200 PG3 600 MW
With limits we get:
m
PGi (20) 1000 PG1 (20) PG 2 (20) PG 3 (20) 1000
i 1
250 100 200 1000
450 MW (compared to 670MW)
m
PGi (30) 1000 300 500 480 1000 280 MW
i 1 21
Lambda-Iteration Limit
Example,contd
Again we continue iterating until the convergence
condition is satisfied.
With limits the final solution of , is 24.43 $/MWh
(compared to 23.53 $/MWh without limits).
Maximum limits will always cause to either increase
or remain the same.
Final solution is:
PG1 (24.43) 300 MW (at maximum limit)
PG 2 (24.43) 443 MW
PG 3 (24.43) 257 MW
22
Back of Envelope Values
$/MWhr = fuelcost * heatrate + variable O&M
Typical incremental costs can be roughly
approximated:
Typical heatrate for a coal plant is 10, modern
combustion turbine is 10, combined cycle plant is 6 to
8, older combustion turbine 15.
Fuel costs ($/MBtu) are quite variable, with current
values around 2 for coal, 3 to 5 for natural gas, 0.5 for
nuclear, probably 10 for fuel oil.
Hydro costs tend to be quite low, but are fuel (water)
constrained
Wind and solar costs are zero.

23
Inclusion of Transmission
Losses
The losses on the transmission system are a
function of the generation dispatch.
In general, using generators closer to the
load results in lower losses
This impact on losses should be included
when doing the economic dispatch
Losses can be included by slightly rewriting
the Lagrangian to include losses PL:

m m
L(PG , ) Ci ( PGi ) PD PL ( PG ) PGi
i 1 i 1 24
Impact of Transmission
Losses
The inclusion of losses then impacts the necessary
conditions for an optimal economic dispatch:
m m
L(PG , ) Ci ( PGi )
PD PL ( PG ) PGi .
i 1 i 1
The necessary conditions for a minimum are now:
L(PG , ) dCi ( PGi ) PL ( PG )
1 0
PGi dPGi PGi
m
PD PL ( PG ) PGi 0
i 1
25
Impact of Transmission
Losses
dCi ( PGi ) PL ( PG )
Solving for , we get: 1 0
dPGi PGi
1 dCi ( PGi )

PL ( PG ) dPGi
1 P
Gi
th
Define the penalty factor Li for the i generator
(don't confuse with Lagrangian L!!!)
1 The penalty factor
Li
PL ( PG ) at the slack bus is
1 P always unity!
Gi 26
Impact of Transmission
Losses
The condition for optimal dispatch with losses is then
L1 IC1 ( PG1 ) L2 IC2 ( PG 2 ) Lm ICm ( PGm )
1
Li . So, if increasing PGi increases
PL ( PG )
1 P
Gi
PL ( PG )
the losses then 0 Li 1.0
PGi
This makes generator i appear to be more expensive
(i.e., it is penalized). Likewise Li 1.0 makes a generator
appear less expensive.
27
Calculation of Penalty
Factors
Unfortunately, the analytic calculation of Li is
somewhat involved. The problem is a small change
in the generation at PGi impacts the flows and hence
the losses throughout the entire system. However,
using a power flow you can approximate this function
by making a small change to PGi and then seeing how
the losses change:
PL ( PG ) PL ( PG ) 1
Li
PGi PGi PL ( PG )
1
PGi 28
Two Bus Penalty Factor
Example

PL ( PG ) PL ( PG ) 0.37 MW
0.0387 0.037
PG 2 PG 2 10 MW
L2 0.9627 L2 0.9643 29
Thirty Bus ED Example
Now consider losses.
Because of the penalty factors the generator in
costs are no longer identical.

30
Area Supply Curve
The area supply curve shows the cost to prod
next MW of electricity, assuming area is econo
dispatched10.00

7.50

Supply
curve for
5.00
thirty bus
system
2.50

0.00
0 100 200 300 400
Total Area Generation (MW)

31
Economic Dispatch -
Summary
Economic dispatch determines the best way to
minimize the current generator operating costs.
The lambda-iteration method is a good approach
for solving the economic dispatch problem:
generator limits are easily handled,
penalty factors are used to consider the impact of
losses.
Economic dispatch is not concerned with
determining which units to turn on/off (this is the
unit commitment problem).
Basic form of economic dispatch ignores the
transmission system limitations.

32
Security Constrained ED
or Optimal Power Flow
Transmission constraints often limit
ability to use lower cost power.
Such limits require deviations from
what would otherwise be minimum
cost dispatch in order to maintain
system security.
Need to solve or approximate power
flow in order to consider transmission
constraints.
33
Security Constrained ED
or Optimal Power Flow
The goal of a security constrained ED or
optimal power flow (OPF) is to determine the
best way to instantaneously operate a power
system, considering transmission limits.
Usually best = minimizing operating cost,
while keeping flows on transmission below
limits.
In three bus case the generation at bus 3 must
be limited to avoid overloading the line from
bus 3 to bus 2.

34
Security Constrained
Dispatch
-22 MW 22 MW
Bus 2 4 MVR -4 MVR Bus 1
1.00 PU
357 MW
179 MVR
1.00 PU
0 MW
100% 37 MVR
-122 MW
194 MW OFF AGC -142 MW
49 MVR 41 MVR 100 MW
232 MVR AVR ON
145 MW 100% 124 MW
-37 MVR -33 MVR
Bus 3 1.00 PU
Home Area
179 MW
Scheduled Transactions 89 MVR
100.0 MW 448 MW AGC ON
19 MVR AVR ON

Need to dispatch to keep line


from bus 3 to bus 2 from overloading
35
Multi-Area Operation
In multi-area system, rules have been
established regarding transactions on tie-lines:
In Eastern interconnection, in principle, up to
nominal thermal interconnection capacity,
In Western interconnection there are more
complicated rules
The actual power that flows through the entire
network depends on the impedance of the
transmission lines, and ultimately determine
what are acceptable patterns of dispatch:
Can result in need to curtail transactions that
otherwise satisfy rules.
Economically uncompensated flow through
other areas is known as parallel path or loop
flows.
Since ERCOT is one area, all of the flows on AC
lines are inside ERCOT and there is no
uncompensated flow on AC lines. 36
Seven Bus Case: One-line
System has
three areas 44 MW -42 MW -31 MW 31 MW 80 MW
1.05 PU
3
0.99 PU
4
30 MVR Top area
1 1.00 PU
62 MW 106 MW -37 MW 110 MW -32 MW
has five
AGC ON 40 MVR
94 MW
buses
Case Hourly Cost AGC ON
38 MW 16933 $/MWH
-14 MW
-61 MW
32 MW
1.04 PU 79 MW -77 MW 1.01 PU
Top Area Cost 5
2
8029 $/MWH
40 MW
40 MW -39 MW 130 MW No net
20 MVR 40 MVR
168 MW AGC ON interchange
-40 MW 20 MW -20 MW
40 MW between
1.04 PU 1.04 PU Any areas.
6 20 MW 7
-20 MW
200 MW 200 MW
Left Area Cost Right Area Cost
Left area 200 MW AGC ON
0 MVR
4189 $/MWH 4715 $/MWH
201 MW AGC ON
0 MVR

has one
Right area has one
bus
bus 37
Seven Bus Case: Area View
Area Losses
Top 7.09 MW

Actual
40.1 MW
flow
-40.1 MW
0.0 MW
0.0 MW between
System has areas
40 MW of
Scheduled
Loop Flow
Left Right flow
Area Losses 40.1 MW Area Losses
0.33 MW 0.0 MW 0.65 MW

Loop flow can result in higher losses


38
Seven Bus - Loop Flow?
Area Losses
Top 9.44 MW

Note that
4.8 MW
0.0 MW
-4.8 MW Tops
0.0 MW
Losses hav
increased
from
7.09MW to
Left Right
9.44 MW
Area Losses 104.8 MW Area Losses
-0.00 MW 100.0 MW 4.34 MW

100 MW Transaction Transaction has


between Left and Right actually
decreased 39

Вам также может понравиться